• 検索結果がありません。

On the zeros of the derivatives of the Riemann zeta function under the Riemann hypothesis (Analytic Number Theory : Distribution and Approximation of Arithmetic Objects)

N/A
N/A
Protected

Academic year: 2021

シェア "On the zeros of the derivatives of the Riemann zeta function under the Riemann hypothesis (Analytic Number Theory : Distribution and Approximation of Arithmetic Objects)"

Copied!
11
0
0

読み込み中.... (全文を見る)

全文

(1)33. 数理解析研究所講究録 第2013巻 2016年 33-43. On the. zeros. of the derivatives of the Riemann zeta function under the Riemann hypothesis. Ade Irma Graduate School of. Suriajaya. Mathematics, Nagoya University Abstract. The number of. zeros. and the distribution of the real part of non‐real. of the. zeros. derivatives of the Riemann zeta function have been. Levin‐. investigated by Berndt, Montgomery, and Akatsuka. Berndt, Levinson, and Montgomery investigated the general case, meanwhile Akatsuka gave sharper estimates for the first derivative of the Riemann zeta function under the truth of the Riemann hypothesis. In this re‐ port, we introduce a generalization of the results of Akatsuka to the k‐th derivative (for positive integer k) of the Riemann zeta function. son,. Introduction. 1. Zeros of the derivatives of the Riemann zeta function. $\zeta$(s). have been studied for. about 80 years. In 1935, Speiser [Spe] showed that the Riemann hypothesis is equivalent to the first derivative of the Riemann zeta function $\zeta$'(s) having no non‐real zeros in. {\rm Re}(s)<1/2. breakthrough in the study of zeros of the Riemann zeta Speiser, Spira [Spi65, Spi70] studied the zero‐free regions of higher order derivatives of the Riemann zeta function, we write $\zeta$^{(k)}(s) to denote the k‐ th derivative of the Riemann zeta function for positive integers k These results encourage further study in the zeros of $\zeta$^{(k)}(s) For example, in 1970, Berndt [Ber] investigated the number of zeros of $\zeta$^{(k)}(s) He [Ber, Theorem] proved that for any positive integer k, function.. .. This result is. Following. a. the work of. .. .. .. N_{k}(T)=\displaystyle \frac{T}{2 $\pi$}\log\frac{T}{4 $\pi$}-\frac{T}{2 $\pi$}+O(\log T) holds, where N_{k}(T) denotes the multiplicity. Furthermore,. with. the. zeros. of. $\zeta$'(s). number of in. zeros. of. $\zeta$^{(k)}(s). with. 1973, Spira [Spi73] hypothesis. In 1974,. and the Riemann. (1.1) 0<{\rm Im}(s)\leq T. ,. counted. also studied the relation between Levinson and. Montgomery. [LM] studied many properties related to the distribution of zeros of $\zeta$^{(k)}(s) , including the location of zeros. They [LM, Theorem 10] also showed that for any positive integer k,. $\zeta$^{(k)}($\rho$^{(k)}=0, <$\gam a$^{(k)\prime}\displayst le\leqT\sum_{$\rho$^{(k)}-$\beta$^{(k)}+i$\gam a$^{(k)}($\beta$^{(k)}-\frac{1}2)=\frac{kT}{2$\pi$}\log\log\frac{T}2$\pi$}+\frac{1}2$\pi$}(\frac{1}2\log2-k\log\log2)T. -k\displaystyle \mathrm{L}\mathrm{i}(\frac{T}{2 $\pi$})+(1.2) O(\log T). 2010 Mathematics. Sub_{2}ect Classification: Primary llM06. Keywords and phrases: Riemann zeta function, derivative, zeros. The original paper [Sur] is to appear in Functiones et Approximatio Commentarii Mathematici. This work was partly supported by Nitori International Scholarship Foundation and Iwatani Naoji Foundation..

(2) 34. holds,. where the. sum. is counted with. multiplicity. and. \displaystyle\mathrm{L}\mathrm{i}(x):=\int_{2}^{x}\frac{dt}{\logt}. The above estimate shows how the real parts of non‐real around the critical line {\rm Re}(s)=1/2 The zeros of $\zeta$^{(k)}(s) .. zeros. of. $\zeta$^{(k)}(s). are. distributed. the critical line. near. were. then. by Conrey [CG] 1996, Yildirim [Yi196, YilOO] investigated non‐real zeros of $\zeta$''(s) and $\zeta$'' (s) in the region to the left of the critical line, that is in the region {\rm Re}(s)<1/2 He succeeded in showing that the Riemann hypothesis implies that $\zeta$''(s) and $\zeta$'' (s) each has only one pair of non‐real zeros in {\rm Re}(s)<1/2 Unfortunately, results analogous to Speiser’s [Spe] were and Ghosh. studied further. in 1989.. In. .. .. Currently,. not obtained.. no. results similar to. Speiser’s [Spe]. are. known for. higher. order. derivatives. In. 2012, Akatsuka [Aka, Theorems. results obtained and. (1.2)). by. for the. Berndt and. case. by. 1 and. 3] improved. each of the. k=1 under the. error. term of the. Montgomery (eq. (1.1) assumption of the truth of the Riemann hypothesis. mentioned above. Levinson and. He showed that. $\zeta$'($\rho$,)=0, <$\gam a$^{i_\leq}T\displaystyle\sum_{$\rho$'=$\beta$'+i$\gam a$'}($\beta$'-\frac{1}{2})=\frac{T}{2$\pi$}\log\log\frac{T}{2$\pi$}+\frac{1}{2$\pi$}(\frac{1}{2}\log2-\log\log2)T. -\displaystyle \mathrm{L}\mathrm{i}(\frac{T}{2 $\pi$})+O( \log\log T)^{2}). and. N_{1}(T)=\displaystyle \frac{T}{2 $\pi$}\log\frac{T}{4 $\pi$}-\frac{T}{2 $\pi$}+O(\frac{\log T}{(\log\log T)^{1/2} ). hold if the Riemann. hypothesis. Akatsuka’s method in the. case. is true. In this. report,. interested in. we are. investigating. when k\geq 2.. Some notation and main results. 2. results, we define some notation. by \mathbb{R} and \mathb {C} the set of all real numbers and the set of all complex numbers, respectively. Throughout this report, the letter k is used as a fixed positive integer, unless otherwise specified. For convenience, we let $\rho$^{(k)}=$\beta$^{(k)}+i$\gamma$^{(k)} Before. wc. introduce. In this report. represent non‐real. Corollary 2, on. of. our. denote. zeros. of. $\zeta$^{(k)}(s). .. results introduced in this report is a generalization of Theorem 1, and Theorem 3 of [Aka], respectively. Note that each sum counts the non‐real. Each of the. zeros. we. $\zeta$^{(k)}(s). following with. multiplicity. and that. O_{k} denotes the. error. terms which. depend only. k.. Theorem 1. Assume that the Riemann. hypothesis. is true.. Then. for. any T>4 $\pi$ ,. have. $\rho$^{(k)}-$\beta$^{(k)}+i$\gam $^{(k)}\displayte\sum_{0<$\gam $^{(k)}\leqT},. ($\beta$^{(k)}-\displaystyle \frac{1}{2})=\frac{kT}{2 $\pi$}\log\log\frac{T}{2 $\pi$}+\frac{1}{2 $\pi$}(\frac{1}{2}\log 2-k\log\log 2)T. we.

(3) 35. -k\displaystyle \mathrm{L}\mathrm{i}(\frac{T}{2 $\pi$})+O_{k}( \log\log T)^{2}) Corollary. 2.. for. (Cf. [LM (where T ,. 0<U<T. Theorem. 3].). Assume that the Riemann. is restricted to. satisfy. T>4 $\pi$ ),. we. hypothesis. is true.. .. Then. have. $\rho^{(k)}=$\beta$^{(k)}+i$\gam $^{(k)}\displayte\sum_{T<$\gam $^{(k)}\leqT+U}, ($\beta$^{(k)}-\displaystyle \frac{1}{2})=\frac{kU}{2 $\pi$}\log\log\frac{T}{2 $\pi$}+\frac{1}{2 $\pi$}(\frac{1}{2}\log 2-k\log\log 2)U. +O(\displaystyle \frac{U^{2} {T\log T})+O_{k}( \log\log T)^{2}). Here the any. error. term. parameters.. o(\displaystyle \frac{U^{2} {T\log T}). holds. uniformly,. Theorem 3. Assume that the Riemann. in other. hypothesis. is true.. it does not. words,. Then. for T\geq 2_{f}. N_{k}(T)=\displaystyle \frac{T}{2 $\pi$}\log\frac{T}{4 $\pi$}-\frac{T}{2 $\pi$}+O_{k}(\frac{\log T}{(\log\log T)^{1/2} ) where. N_{k}(T). We write the Riemann. is. defined. as. {\rm Re}(s). and. hypothesis. in. we. on. have. ,. equation (1.1).. {\rm Im}(s) (for as. depend. .. RH,. any s\in \mathbb{C} ). and. finally,. we. as. and t. $\sigma$. respectively. We. define two functions. F(s). abbreviate. and. G_{k}(s). ,. as. follows:. F(s):=2^{s}$\pi$^{s-1}\displaystyle \sin(\frac{ $\pi$ s}{2}) $\Gamma$(1-s) , G_{k}(s):=(-1)^{k}\frac{2^{s} {(\log 2)^{k} $\zeta$^{(k)}(s) By. the above definition of. F(s). ,. we can. check. easily. that the functional. .. equation for $\zeta$(s). states. $\zeta$(s)=F(s) $\zeta$(1-s). Sketch of. 3. In this report,. [Sur]. we. .. proofs mainly give only. sketch of the. proofs. Refer. 3.1. Key. original. paper. lemmas. We first introduce. a. few lemmas and. propositions which. [Aka]. Lemma 3.1. There exists. an. a_{k}\geq 10 such that. |G_{k}(s)-1|\displaystyle \leq\frac{1}{2}(\frac{2}{3})^{ $\sigma$/2} holds. to the. for the details.. for. any. $\sigma$\geq a_{k}.. are. analogues of. those in.

(4) 36. Proof.. [LM, inequality (3.2) (p.. See. Lemma 3.2. There exists. a. \square. 54. $\sigma$_{k}\leq-1 such that. |\displayst le\sum_{j=1}^{k}\left(\begin{ar y}{l k\ j \end{ar y}\right)(-1)^{j-1}\frac{1}\frac{F(k)}{F(k-,)}(s)}\frac{$\zeta$^{(j)}{$\zeta$}(1-s)|\leq2^{$\sigma$} holds in the. region $\sigma$\leq$\sigma$_{k}, t\geq 2.. Proof. (Sketch) begin by estimating. We. in the. \displaystyle \frac{F^{(k)} {F(k-j)}(s) (j=1,2, \cdots, k). region $\sigma$<1, t\geq 2 Using methods similar .. to. [LM,. pp.. 54−55], we can show that (i.e. sufficiently large. for any positive integer k , we can take $\sigma$_{k_{1}}\leq-1 sufficiently small in the negative direction) so that for any s with $\sigma$\leq$\sigma$_{k_{1} and t\geq 2 ,. |\displaystyle \frac{F^{(k)} {F(k-j)}(s)|\geq\frac{1}{2k}(\log(1- $\sigma$) ^{j} Next. we. (3.1). .. estimate. \displaystyle \frac{$\zeta$^{(j)} { $\zeta$}(1-s) (j=1,2, \cdots, k) In the. have. we. region $\sigma$\leq-1, t\geq 2. ,. we can. make. use. .. of the Dirichlet series of. $\zeta$(s). and. $\zeta$^{(j)}(s). to. obtain. Now any. |\displaystyle\frac{$\zeta$^{(j)}{$\zeta$}(1-s)|\leq\frac{2^{$\sigma$}{2-\frac{$\pi$^{2}{6}(\frac{1}{2}(\log2)^{j}+\sum_{l=0}^{j}\frac{(\log2)^{j-l}\frac{j^1}{(j-l)!}{(-$\sigma$)^{l+1}\mathrm{I}. combining inequalities (3.1). and. (3.2),. for. $\sigma$\leq$\sigma$_{k_{1} and t\geq 2. ,. and. (3.2) noting that for. positive integer k,. \displayst le\lim_{$\sigma$\rightar ow-\infty}\frac{2k}{2-\frac{$\pi$^{2}{6}\sum_{j=1}^{k}\left(\begin{ar ay}{l k\ j \end{ar ay}\right)\frac{1}(\log(1-$\sigma$)^{j}(\frac{1}2(\log2)^{j}+\sum_{l=0}^{j}\frac{(\log2)^{j-l}\frac{j!}(j-l)^{1} {(-$\sigma$)^{l+1}\mathrm{I}=0, we can. find. $\sigma$_{k}\leq$\sigma$_{k_{1}}(\leq-1). such that. \displaystyle\frac{2k}{2-\frac{$\pi$^{2}{6}\sum_{j=1}^{k}\left(\begin{ar ay}{l k\ j \end{ar ay}\right)\frac{1}{(\log(1-$\sigma$)^{j}(\frac{1}{2}(\log2)^{j}+\sum_{l=0}^{j}\frac{(\log2)^{j-l}\frac{j!}(j-l)^{1} {(-$\sigma$)^{l+1})\leq1 holds for any $\sigma$\leq$\sigma$_{k} Now. we. .. This. implies. our. fix the above a_{k} and $\sigma$_{k} to show the. Lemma 3.3. Assume RH Then there exists .. conditions. \square. lemma.. are. satisfied:. a. following. lemma.. t_{k}\displaystyle \geq\max\{a_{k}^{2}, -$\sigma$_{k}\}. such that the. following.

(5) 37. 1. For any. s. satisfying $\sigma$_{k}\leq $\sigma$\leq 1/2. and t\geq t_{k}-1,. |\displaystyle \frac{F^{(k)} {F}(s)|\geq 1 holds. it is. Furthermore, we can holomorphic there and. take. a. branch. of \log(F^{(k)}/F)(s). in that. region such that. in that. region such that. \displaystyle \frac{$\alpha$_{k} $\pi$}{6}<\arg\frac{F^{(k)} {F}(s)<\frac{$\beta$_{k} $\pi$}{6} holds, where. ($\alpha$_{k},$\beta$_{k})=\left\{ begin{ar ay}{l (5,7)ifkisod ,\ (-1, )ifkisev n. \end{ar ay}\right. 2. For any. s. satisfying $\sigma$_{k}\leq $\sigma$<1/2. and. t\geq t_{k}-1_{f}. \displayst le\frac{$\zeta$^{(k)}{$\zeta$}(s)\neq0 holds. it is. Furthermore, we can take holomorphic there and. branch. a. of \log($\zeta$^{(k)}/ $\zeta$)(s). \displaystyle\frac{k$\pi$}{2}<\arg\frac{$\zeta$^{(k)} {$\zeta$}(s)<\frac{3k$\pi$}{2} holds. 3. For all $\sigma$\in \mathbb{R} ,. we. have. $\zeta$( $\sigma$+it_{k})\neq 0, $\zeta$^{(k)}( $\sigma$+it_{k})\neq 0. Proof. (Sketch) To prove condition 1, we apply Stirling’s formula and methods similar to the inequality (3.1). We can show that. proof. F^{(k)}(s)=F(s)(- \displaystyle \log(1-s)+O(1))^{k}(1+O(\frac{1}{|\log s|^{2} )) holds in the. t_{k_{1}}\geq 100. holds for. region $\sigma$_{k}\leq $\sigma$\leq 1/2, t\geq 99. .. Thus for any. integer k\geq 1. ,. we can. of. (3.3) take. some. such that. $\sigma$_{k}\leq $\sigma$\leq 1/2. and. We note from equation $\sigma$_{k}\leq $\sigma$\leq 1/2 and t\geq 99 .. sufficiently large. such that. |\displaystyle \frac{F^{(k)} {F}(s)|\geq 1. (3.4). t\geq t_{k_{1}}-1.. (F^{(k)}/F)(s)=(-1)^{k}(\log t)^{k}+O((\log t)^{k-1}) when Consequently, for odd integer k\geq 1 we can find t_{k_{2}}'\geq 100. (3.3). that. ,. \displaystyle \frac{5 $\pi$}{6}<\arg\frac{F^{(k)} {F}(s)<\frac{7 $\pi$}{6}.

(6) 38. $\sigma$_{k}\leq $\sigma$\leq 1/2. holds for. t\geq t_{k_{2}}'-1. and. .. Similarly,. when k is even,. we. can. also find. such that. t_{k_{2}}''\geq 100 large enough. -\displaystyle \frac{ $\pi$}{6}<\arg\frac{F^{(k)} {F}(s)<\frac{ $\pi$}{6} holds for. $\sigma$_{k}\leq $\sigma$\leq 1/2. (F^{(k)}/F)(s) \log(F^{(k)}/F)(s). has. \mathbb{R},. poles holomorphic. is. t\geq t_{k_{2}}''-1. and. in the. Since all. .. poles of F(s) lie on inequality (3.4) implies that and. zeros. with. This. along region with this branch. We. in t>0. no. .. set. ($\alph$_{k},$\beta$_{k}):=\left{\begin{ar y}{l (5,7)&\mathrm{i}\ athrm{f}k\mathrm{i}\ athrm{s}\mathrm{o}\mathrm{d}\mathrm{d},\ (-1,)&\mathrm{i}\ athrm{f}\ athrm{k}\mathrm{i}\ athrm{s}\mathrm{e}\mathrm{v}\mathrm{e}\mathrm{n}; \end{ar y}\right. and. t_{k2}:=\left{begin{ary}l t_{k2}',&\mathr{i}\mathr{f}k\mathr{i}\mathr{s}\mathr{o}\mathr{d}\mathr{d},\ t_{k2}',&\mathr{i}\mathr{f}k\mathr{i}\mathr{s}\mathr{e}\mathr{v}\mathr{e}\mathr{n}. \end{ary}\ight. From the above. we. calculations,. find that. we. \displaystyle \max\{t_{k_{1} , t_{k_{2} , a_{k}^{2}, -$\sigma$_{k}\}. is. a. candidate for t_{k}.. have proven that t_{k}\displaystyle \geq\max\{a_{k}^{2}, -$\sigma$_{k}\} for which condition 1 holds exists. Since want t_{k} to also satisfy conditions 2 and 3, we need to examine those conditions to. Thus. we. completely. prove the existence of. t_{k}.. To prove condition 2, we first make use of the finiteness of the number of non‐real zeros of $\zeta$^{(j)}(s) in the region $\sigma$<1/2 under RH for any positive integer j (cf. [LM, Corollary. of Theorem 7. (p.. 51. to find. some. t_{k_{3} such. that for all. j=1,2,. \cdots,. k,. $\zeta$^{(j)}(s)\neq 0 in the. region $\sigma$<1/2, t\geq t_{k_{3}}-1.. Next. we. show that. 1/2, t\geq t_{k_{4}}-1. for. we. some. can. take. t_{k_{4}}\geq 100. ,. a. so. branch of that it is. we. have. (3.5). \log($\zeta$^{(k)}/ $\zeta$)(s). holomorphic. in the. region $\sigma$_{k}\leq $\sigma$<. there and. \displaystyle\frac{k$\pi$}{2}<\arg\frac{$\zeta$^{(k)} {$\zeta$}(s)<\frac{3k$\pi$}{2} holds there. by making. use. of the. following inequality. {\rm Re}(\displaystyle \frac{$\zeta$^{(j)} {$\zeta$^{(j-1)} (s) \leq-\frac{2}{9}\log|s+O_{$\sigma$_{k} (1) which holds for any j=1 , 2, \cdots, k when $\sigma$_{k}\leq $\sigma$<1/2 and 64−65]). We omit details of the proof here. We then have Now. we. Since. set. t_{k_{5}. we are. According we. \displaystyle \max\{t_{k_{1} , t_{k_{2} , t_{k_{3} , t_{k_{4} , a_{k}^{2}, -$\sigma$_{k}\}. to. as a. t\geq t_{k_{3}}-1 (see [LM,. pp.. candidate for t_{k}.. :=\displaystyle \max\{t_{k_{1}}, t_{k_{2}}, t_{k_{3}}, t_{k_{4}}, a_{k}^{2}, -$\sigma$_{k}\}. assuming RH, $\zeta$( $\sigma$+it)\neq 0 for. [Spi65,. Table 1. (p. 678). any t>0 if. and Theorem. 1],. $\sigma$\neq 1/2. for any. have. $\zeta$^{(k)}( $\sigma$+it)\neq 0 ( $\sigma$\geq 7k/4+2, t\in \mathbb{R}). .. positive integer k,.

(7) 39. Since. Hence,. t_{k_{5}}\geq t_{k_{3}} from (3.5),. we. ,. for any. positive integer k. $\zeta$(1/2+it_{k})\neq 0. ,. $\zeta$^{(k)}( $\sigma$+it)\neq 0. have. we. only. need to find. and. t\geq t_{k_{5}}.. t_{k}\in[t_{k_{5}}+1, t_{k_{5}}+2]. $\zeta$^{(k)}( $\sigma$+it_{k})\neq 0. and. $\sigma$<1/2. for. for. for which. 1/2\leq $\sigma$\leq 7k/4+2. hold.. Thus,. we. have shown that t_{k} defined above satisfies. t_{k}\displaystyle \geq\max\{a_{k}^{2}, -$\sigma$_{k}\}. and also. conditions 1 to 3.. We. now. Now. we. \square. fix a_{k}, $\sigma$_{k} , and t_{k} which. give. two bounds for—. satisfy. Lemmas. 3.1, 3.2, and 3.3.. \arg $\zeta$( $\sigma$+iT)+\arg G_{k}( $\sigma$+iT). .. We. use. methods similar. 2.3, 2.4, 2.6]. We take the logarithmic branches of \log $\zeta$(s) and \log G_{k}(s) [Aka, such that they tend to 0 as $\sigma$\rightarrow\infty and are holomorphic in \mathbb{C}\backslash \{ $\rho$+ $\lambda$| $\zeta$( $\rho$)=0 or \infty, $\lambda$\leq 0\} and \mathb {C}\backslash { $\rho$^{(k)}+ $\lambda$|$\zeta$^{(k)}($\rho$^{(k)})=0 or \infty, $\lambda$\leq 0 }, respectively. We write Lemmas. to. ‐. \displaystyle \arg $\zeta$( $\sigma$+iT)+\arg G_{k}( $\sigma$+iT)=\arg\frac{G_{k} { $\zeta$}( $\sigma$+iT). ,. where the argument on the right hand side is determined so that \log(G_{k}/ $\zeta$)(s) tends to 0 as $\sigma$\rightarrow\infty and is holomorphic in \mathb {C}\backslash { z+ $\lambda$|($\zeta$^{(k)}/ $\zeta$)(z)=0 or \infty, $\lambda$\leq 0 }. Lemma 3.4. Assume RH and let T\geq t_{k}. (since T\geq t_{k}\geq 100, $\epsilon$_{0}<1/8). ,. we. have. Then for any $\epsilon$_{0}>0 satisfying $\epsilon$_{0}<(2\log T)^{-1} for 1/2+$\epsilon$_{0}< $\sigma$\leq a_{k}, .. \displaystyle\arg\frac{G_{k}{$\zeta$}($\sigma$+iT)=O_{a_{k},t_{k}(\frac{\log\frac{\logT}{$\epsilon$_{0} {$\sigma$-\frac{1}2-$\epsilon$_{0}) We omit the. proof. of the above lemma. Lemma 3.5. Assume RH and let. (refer. to. [Sur,. Lemma. A\geq 2 be fixed. Then there. 2.3]).. exists. a. constant. C_{0}>0. such that. |$\zeta$^{(k)}( $\sigma$+it)|\displaystyle \leq\exp(C_{0}(\frac{(\log T)^{2(1- $\sigma$)} {\log\log T}+(\log T)^{1/10}). holds. for T\geq t_{k}, T/2\leq t\leq 2T, 1/2-(\log\log T)^{-1}\leq $\sigma$\leq A.. Proof Referring. to. [Tit, (14.14.2), (14.14.5). and the first. equation. on. p.. 384],. we. can. show that. holds for. [Aka,. pp.. | $\zeta$( $\sigma$+it)|\displaystyle \leq\exp(C_{1}(\frac{(\log T)^{2(1- $\sigma$)} {\log\log T})+(\log T)^{1/10}). 1/2-2(\log\log T)^{-1}\leq $\sigma$\leq A+1, T/3\leq t\leq 3T. for. some. constant. 2251−2252]).. Applying Cauchy’s integral formula,. we see. (3.6) C_{1}>0 (cf.. that. $\zeta$^{(k)}(s)=\displaystyle \frac{k!}{2 $\pi$ i}\int_{|z-s|= $\epsilon$}\frac{ $\zeta$(z)}{(z-s)^{k+1} dz. for. 0< $\epsilon$<1/2. region defined by 1/2-(\log\log T)^{-1}\leq $\sigma$\leq A and T/2\leq t\leq 2T Applying inequality (3.6) and by taking $\epsilon$=(2(\log\log T)^{1/k})^{-1}(<1/2) we obtain Lemma 3.5. \square holds in the. .. ,.

(8) 40. Lemma 3.6. Assume RH and let. T\geq t_{k}. Then. .. for. any. 1/2\leq $\sigma$\leq 3/4. \displaystyle \mathrm{a}x\mathrm{g}G_{k}( $\sigma$+iT)=O_{a_{k} (\frac{(\log T)^{2(1- $\sigma$)} {(\log\log T)^{1/2} ) The. Proof. [Aka,. proof proceeds. 2252−2253] [Aka].. pp.. 2.6 of. in the. same. for the detailed. way. proof. as. the. and. proof. use. not. are. essential,. .. of Lemma 2.4 of. Lemma 3.5 above in. [Aka].. Refer to. place of Lemma \square. Remark 1. The restrictions of the lower bound of T 3.6. have. we. ,. but. they. sufficient for. are. our. we. gave in Lemmas. 3.4, 3.5, and. needs.. Proof of Theorem 1. 3.2. The. following proposition gives. the main term of Theorem 1.. Proposition 3.7. Assume RH Take a_{k} and t_{k} which satisfy Lemmas 3.1 and 3.3 respec‐ tively. Then for T\geq t_{k} which satisfies $\zeta$^{(k)}( $\sigma$+iT)\neq 0 and $\zeta$( $\sigma$+iT)\neq 0 for any $\sigma$\in \mathbb{R}, .. have. we. $\rho$^{(k)}-$\beta$^{(k)}+i$\gam $^{(k)}\displayt e\sum_{0<$\gam $^{(k)}\leqT}, ($\beta$^{(k)}-\displaystyle \frac{1}{2})=\frac{kT}{2 $\pi$}\log\log\frac{T}{2 $\pi$}+\frac{1}{2 $\pi$}(\frac{1}{2}\log 2-k\log\log 2)T-k\mathrm{L}\mathrm{i}(\frac{T}{2 $\pi$}). +\displaystyle \frac{1}{2 $\pi$}\int_{1/2}^{a_{k} (- \arg $\zeta$( $\sigma$+iT)+\arg G_{k}( $\sigma$+iT) d $\sigma$+O_{k}(1). where the. Lemma. logarithmic. are. taken. as. in Section 3.1. (see. the. paragraph preceding. 3.4).. We omit the as. branches. ,. proof (refer. to. [Sur, Proposition 2.2]).. The. proof. of Theorem 1 is done. follows. First of. all,. for any $\sigma$\in \mathbb{R}. .. we. consider for. From Lemma. T\geq t_{k} which satisfies 3.4, we have. $\zeta$^{(k)}( $\sigma$+iT)\neq 0. and. $\zeta$( $\sigma$+iT)\neq 0. \displaystyle\int_{1/2+ $\epsilon$_{0}^{a_{k}\arg\frac{G_{k}{$\zeta$}($\sigma$+iT)d$\sigma$\l_{a_{k},t_{k}\int_{1/2+ $\epsilon$_{0}^{a_{k}\frac{\log\frac{\logT}{$\epsilon$_{0} {$\sigma$-\frac{1}2-$\epsilon$_{0}d$\sigma$\l_{a_{k}\log\frac{\logT}{$\epsilon$_{0}\log\frac{1}$\epsilon$_{0}. Next, from Lemma 3.6,. \displaystyle \arg G_{k}( $\sigma$+iT)=O_{a_{k} (\frac{(\log T)^{2(1- $\sigma$)} {(\log\log T)^{1/2} ) equation (2.23) of [Aka, implies that. and from. RH. holds. uniformly. for. p.. for. 1/2\leq $\sigma$\leq 3/4. 2251] (cf. [Tit, equations (14.14.3). \displaystyle \arg $\zeta$( $\sigma$+iT)=O(\frac{(\log T)^{2(1- $\sigma$)} {\log\log T}). 1/2\leq $\sigma$\leq 3/4 Thus, .. \displaystyle\int_{1/2}^{1/2+2$\epsilon$_{0} \arg\frac{G_{k} {$\zeta$}($\sigma$+iT)d$\sigma$\l_{a_{k} \frac{\logT}{(\log\logT)^{1/2} $\epsilon$_{0}.. and. (14.14.5)]),.

(9) 41. Now. we. take. $\epsilon$_{0}=(4\log T)^{-1}(<(2\log T)^{-1}). ,. then. we. have. \displaystyle \int_{1/2}^{a_{k} \arg\frac{G_{k} { $\zeta$}( $\sigma$+iT)d $\sigma$\l _{a_{k},t_{k} (\log\log T)^{2} Applying this only. k,. on. to. Proposition 3.7 and noting that. a_{k}. and t_{k}. are. fixed constants that. depend. have. we. $\rho$^{(k)}-$\beta$^{(k)}+i$\gam $^{(k)}\displayt e\sum_{0<$\gam $^{(k)}\leqT}, ($\beta$^{(k)}-\displaystyle \frac{1}{2})=\frac{kT}{2 $\pi$}\log\log\frac{T}{2 $\pi$}+\frac{1}{2 $\pi$}(\frac{1}{2}\log 2-k\log\log 2)T-k\mathrm{L}\mathrm{i}(\frac{T}{2 $\pi$}) +O_{k}((\log\log T)^{2}). For. 4 $\pi$<T<t_{k}. thus. ,. we are. k. depend only For T\geq t_{k} such that on. so. adding. this. can. some. finite number of terms which. be included in the. $\zeta$^{(k)}( $\sigma$+iT)=0. error. $\zeta$( $\sigma$+iT)=0. or. depend. on. .. (3.7). t_{k} and ,. term.. for. some. $\sigma$\in \mathbb{R} , there is. some. increment in the value of. as. much. $\rho$^{(k)}-$\beta$^{(k)}+i$\gam a$^{(k)}\displaystle\sum_{0<$\gam a$^{(k)}\leqT},($\beta$^{(k)}-\frac{1}2) $\rho$^{(k)}-$\beta$^{(k)}+i$\gam a$^{(k)}\displaystle\sum_{$\gam a$^{(k)}=T,($\beta$^{(k)}-\frac{1}2). as. We estimate this and show that this. can. be included in the. We start for any. .. by taking a small 0< $\epsilon$<1 such that $\sigma$\in \mathbb{R} According to equation (3.7),. term of. error. $\zeta$^{(k)}( $\sigma$+i(T\pm $\epsilon$))\neq 0. and. equation (3.7).. $\zeta$( $\sigma$+i(T\pm $\epsilon$))\neq 0. .. $\rho^{(k)}=$\beta$^{(k)}+i$\gam $^{(k)}\displayte\sum_{0<$\gam $^{(k)}\leqTpm$\epsilon$},. ($\beta$^{(k)}-\displaystyle \frac{1}{2})=\frac{k(T\pm $\epsilon$)}{2 $\pi$}\log\log\frac{T\pm $\epsilon$}{2 $\pi$}+\frac{1}{2 $\pi$}(\frac{1}{2}\log 2-k\log\log 2)(T\pm $\epsilon$) -k\displaystyle \mathrm{L}\mathrm{i}(\frac{T\pm $\epsilon$}{2 $\pi$})+O_{k}( \log\log T)^{2}). .. Thus,. $\rho$^{(k)}-$\beta$^{(k)}+i$\gam a$^{(k)}\displayst le\sum_{T-$\epsilon$< \gam a$^{(k)}\leqT+$\epsilon$'}($\beta$^{(k)}-\frac{1}2)=\frac{k$\epsilon$}{$\pi$}\log\log\frac{T}2$\pi$}+\frac{$\epsilon$}{$\pi$}(\frac{1}2\log2-k\log\log2)+O_{k}(\log\logT)^{2}) =O_{k}((\log\log T)^{2}). This. .. implies. Therefore,. $\rho$^{(k)}-$\beta$^{(k)}+i$\gam a$^{(k)}\displaystyle\sum_{$\gam a$^{(k)}=T},($\beta$^{(k)}-\frac{1}2)=O_{k}(\log\logT)^{2}). this increment. can. also be included in the. error. .. term and the. proof is complete. \square.

(10) 42. Proof of. 3.3. This is. an. Corollary. 2. immediate consequence of Theorem 1. See. Section 3. [LM,. (the ending part. p. 58. of \square. Proof of Theorem 3. 3.4. Finally, we give the proof of Theorem 3. give the main term of our estimate.. We first introduce the. following proposition. which. Proposition 3.8. Assume RH Take t_{k} for T\geq 2 which satisfies $\zeta$( $\sigma$+iT)\neq 0. which. .. and. of Lemma. all conditions. satisfies. $\zeta$^{(k)}( $\sigma$+iT)\neq 0 for. all $\sigma$\in \mathbb{R} ,. we. 3.3. Then. have. N_{k}(T)=\displaystyle \frac{T}{2 $\pi$}\log\frac{T}{4 $\pi$}-\frac{T}{2 $\pi$}+\frac{1}{2 $\pi$}\arg G_{k}(\frac{1}{2}+iT)+\frac{1}{2 $\pi$}\arg $\zeta$(\frac{1}{2}+iT)+O_{k}(1) where the arguments. are. determined. proof (refer. We omit the. to. Proposition. in. as. ,. 3. 7.. [Sur, Proposition 3.1]).. The. of Theorem 3 is. proof. as. follows.. Firstly any $\sigma$\in \mathbb{R}. we. $\zeta$^{(k)}( $\sigma$+iT)\neq 0. consider for T\geq 2 which satisfies Lemma 3.6,. and. $\zeta$( $\sigma$+iT)\neq 0. for. By. .. \displaystyle \arg G_{k}(\frac{1}{2}+iT)=O_{a_{k} (\frac{\log T}{(\log\log T)^{1/2} ) and. again from equation (2.23) of [Aka,. 2251],. p.. have. we. \displaystyle \arg $\zeta$(\frac{1}{2}+iT)=O(\frac{\log T}{\log\log T}) Substituting. these into. Proposition 3.8,. we. .. obtain. N_{k}(T)=\displaystyle \frac{T}{2 $\pi$}\log\frac{T}{4 $\pi$}-\frac{T}{2 $\pi$}+O_{k}(\frac{\log T}{(\log\log T)^{1/2} ) Next, a. if. $\zeta$( $\sigma$+iT)=0. or. small 0< $\epsilon$<1 such that. in the. of. proof. N_{k}(T). $\zeta$^{(k)}( $\sigma$+iT)=0 for $\zeta$^{(k)}( $\sigma$+i(T\pm $\epsilon$))\neq 0. of Theorem 1. Then. can. be included in the. (T\geq 2) then again we take $\zeta$( $\sigma$+i(T\pm $\epsilon$))\neq 0 for any $\sigma$\in \mathbb{R} as. $\sigma$\in \mathbb{R}. some. similarly, error. and. we can. .. ,. show that the increment of the value. term of the above. equation which completes the. proof.. \square. References [Aka]. H.. zeros. [Ber]. Akatsuka, Conditional estimates for error terms related to of $\zeta$'(s) J. Number Theory 132 (2012), no. 10, 2242‐2257.. B. C.. 577‐580.. the distribution. of. ,. Berndt,. The number. of. zeros. for. $\zeta$^{(k)}(s). ,. J. Lond. Math. Soc.. (2). 2. (1970),.

(11) 43. [CG]. J. B.. Conrey and A. Ghosh, Zeros of derivatives of the Riemann zeta‐function near line, in “‘Analytic number theory, Proc. Conf. in Honor of P. T. Bateman B. C. Berndt et al. (eds.), Progr. Math. Vol. 85, (Allerton Park, IL, USA, 1989 Birkhäuser Boston, Boston, MA, 1990, 95‐110. the critical. ,. [LM]. N. Levinson and H. L.. function, Acta. [Spe]. Montgomery,. (1974),. Math. 133. A.. Speiser, Geometrisches 1, 514‐521.. no.. [Spi70]. R.. zur. Zeros. of. the derivatives. of. the Riemann zeta‐. 49‐65.. Riemannschen. Math. Ann. 110. (1935),. Proc. Amer. Math. Soc. 26. (1970),. Zetafunktion,. Spira, Another zero‐free region for $\zeta$^{(k)} (s). ,. 246‐247.. [Spi65]. R.. Spira, Zero‐free regions of $\zeta$^{(k)} (s),. [Spi73]. R.. Spira,. of $\zeta$'(s). Zeros. J. Lond. Math. Soc. 40. and the Riemann. hypothesis,. (1965),. 677‐682.. Illinois J. Math. 17. (1973),. 147‐152.. [Sur]. A. I.. On the. zeros of the k‐th derivative of the Riemann zeta function hypothesis, Preprint, arxiv:1310.6489 [math.NT] (to appear in Approximatio Commentarii Mathematici).. Suriajaya,. under the Riemann Functiones et. [Tit]. E. C.. D. R.. [Yi196] no.. Titchmarsh,. Heath‐Brown),. The. theory of the Riemann zeta‐function, Press, 1986.. C. Y. Yildirim, A Note 8, 2311‐2314.. [YilOO]. C. Y.. Yildirim,. Zeros. on. $\zeta$''(s). and. $\zeta$'' (s). of $\zeta$''(s) & $\zeta$'' (s). in. 89‐108.. Graduate School of Mathematics. Nagoya University Furo‐cho, Chikusa‐ku, Nagoya. 464‐8602. JAPAN E‐‐mail address:. second ed.. (revised by. Oxford Univ.. m12026a@math.nagoya‐u.ac.jp. ,. Proc. Amer. Math. Soc. 124. $\sigma$<\displaystyle \frac{1}{2}. ,. Turk. J. Math. 24. (1996),. (2000),. no.. 1,.

(12)

参照

関連したドキュメント

If condition (2) holds then no line intersects all the segments AB, BC, DE, EA (if such line exists then it also intersects the segment CD by condition (2) which is impossible due

The inclusion of the cell shedding mechanism leads to modification of the boundary conditions employed in the model of Ward and King (199910) and it will be

In this paper we develop a general decomposition theory (Section 5) for submonoids and subgroups of rings under ◦, in terms of semidirect, reverse semidirect and general

On the other hand, when M is complete and π with totally geodesic fibres, we can also obtain from the fact that (M,N,π) is a fibre bundle with the Lie group of isometries of the fibre

It is suggested by our method that most of the quadratic algebras for all St¨ ackel equivalence classes of 3D second order quantum superintegrable systems on conformally flat

This paper develops a recursion formula for the conditional moments of the area under the absolute value of Brownian bridge given the local time at 0.. The method of power series

Answering a question of de la Harpe and Bridson in the Kourovka Notebook, we build the explicit embeddings of the additive group of rational numbers Q in a finitely generated group

Then it follows immediately from a suitable version of “Hensel’s Lemma” [cf., e.g., the argument of [4], Lemma 2.1] that S may be obtained, as the notation suggests, as the m A